www.vorhilfe.de
Vorhilfe

Kostenlose Kommunikationsplattform für gegenseitige Hilfestellungen.
Hallo Gast!einloggen | registrieren ]
Startseite · Forum · Wissen · Kurse · Mitglieder · Team · Impressum
Forenbaum
^ Forenbaum
Status Englisch
  Status Grammatik
  Status Lektüre
  Status Korrekturlesen
  Status Übersetzung
  Status Sonstiges (Englisch)

Gezeigt werden alle Foren bis zur Tiefe 2

Navigation
 Startseite...
 Neuerdings beta neu
 Forum...
 vorwissen...
 vorkurse...
 Werkzeuge...
 Nachhilfevermittlung beta...
 Online-Spiele beta
 Suchen
 Verein...
 Impressum
Das Projekt
Server und Internetanbindung werden durch Spenden finanziert.
Organisiert wird das Projekt von unserem Koordinatorenteam.
Hunderte Mitglieder helfen ehrenamtlich in unseren moderierten Foren.
Anbieter der Seite ist der gemeinnützige Verein "Vorhilfe.de e.V.".
Partnerseiten
Weitere Fächer:

Open Source FunktionenplotterFunkyPlot: Kostenloser und quelloffener Funktionenplotter für Linux und andere Betriebssysteme
Forum "Funktionen" - Taylorformel
Taylorformel < Funktionen < eindimensional < reell < Analysis < Hochschule < Mathe < Vorhilfe
Ansicht: [ geschachtelt ] | ^ Forum "Funktionen"  | ^^ Alle Foren  | ^ Forenbaum  | Materialien

Taylorformel: bestimmter Fehler?
Status: (Frage) beantwortet Status 
Datum: 15:26 Fr 17.06.2011
Autor: bandchef

Aufgabe
Bestimmen Sie für $x [mm] \in [/mm] [ 0,1 ]$ mit Hilfe der Taylorformel einen Näherungswert für die Funktion [mm] $e^x$ [/mm] der um höchstens $5 [mm] \cdot 10^{-5}$ [/mm] vom wahren Wert abweicht.



Die Taylor-Formel lautet ja so:

[mm] $T_n(x) [/mm] = [mm] \sum_{k=0}^n {f^{(k)}(x_0) \over k!}(x-x_0) )^k [/mm] = [mm] f(x_0) [/mm] ) + [mm] \frac{f'(x_0) )}{1!}(x-x_0) [/mm] ) + [mm] \frac{f''(x_0) )}{2!}(x-x_0) )^2 [/mm] + [mm] \ldots [/mm] + [mm] \frac{f^{(n)}(x_0) )}{n!}(x-x_0) )^n$ [/mm]

Mir is in der Formel soweit alles klar, aber ich weiß immer nicht was hier nun in [mm] $x_0$ [/mm] einsetzen soll. Kann man das hier aus der Aufgabenstellung heraus ablesen? Aus dem Intervall? Welche Grenze des Intervalls?


Bei der Aufgabe soll ich ja die Funktion [mm] $e^x$ [/mm] durch die Taylorformel soweit annähern das im obigen Intervall nur noch eine Abweichung von max. $5 [mm] \cdot 10^{-5}$ [/mm] besteht. Wie gehe ich da nun vor? Soll man das ausprobieren wieviel Ableitungen da man da dann braucht? Oder gibt's eine elegantere Möglichkeit?



        
Bezug
Taylorformel: Frage (beantwortet)
Status: (Frage) beantwortet Status 
Datum: 15:47 Fr 17.06.2011
Autor: bandchef

Wenn man nun die gegebene Funktion [mm] $f(x)=e^x$ [/mm] ableitet, dann fällt auf, dass gilt:

[mm] $f(x)=e^x$ [/mm]
[mm] $f'(x)=e^x$ [/mm]
[mm] $f''(x)=e^x$ [/mm]
.
.
.
[mm] $f^{(k)}(x)=e^x$ [/mm]

Löst das evtl. mein Problem? Was ich damit aber nun anfangen soll versteh ich trotzdem nich so wirklich

Bezug
                
Bezug
Taylorformel: Antwort
Status: (Antwort) fertig Status 
Datum: 16:59 Fr 17.06.2011
Autor: MathePower

Hallo bandchef,

> Wenn man nun die gegebene Funktion [mm]f(x)=e^x[/mm] ableitet, dann
> fällt auf, dass gilt:
>  
> [mm]f(x)=e^x[/mm]
>  [mm]f'(x)=e^x[/mm]
>  [mm]f''(x)=e^x[/mm]
>  .
>  .
>  .
>   [mm]f^{(k)}(x)=e^x[/mm]
>  
> Löst das evtl. mein Problem? Was ich damit aber nun
> anfangen soll versteh ich trotzdem nich so wirklich


So geht man vor, wenn man das Taylorpolynom erstellt.

Bleibt noch die Frage nach dem Entwicklungspunkt [mm]x_{0}[/mm].

Natürlich kannst Du die Abschätzung des Restgliedes vom
Entwicklungspunkt [mm]x_{0}[/mm] abhängig machen.


Gruss
MathePower

Bezug
        
Bezug
Taylorformel: Antwort
Status: (Antwort) fertig Status 
Datum: 16:50 Fr 17.06.2011
Autor: MathePower

Hallo bandchef,

> Bestimmen Sie für [mm]x \in [ 0,1 ][/mm] mit Hilfe der Taylorformel
> einen Näherungswert für die Funktion [mm]e^x[/mm] der um
> höchstens [mm]5 \cdot 10^{-5}[/mm] vom wahren Wert abweicht.
>  
>
> Die Taylor-Formel lautet ja so:
>  
> [mm]T_n(x) = \sum_{k=0}^n {f^{(k)}(x_0) \over k!}(x-x_0) )^k = f(x_0) ) + \frac{f'(x_0) )}{1!}(x-x_0) ) + \frac{f''(x_0) )}{2!}(x-x_0) )^2 + \ldots + \frac{f^{(n)}(x_0) )}{n!}(x-x_0) )^n[/mm]
>  
> Mir is in der Formel soweit alles klar, aber ich weiß
> immer nicht was hier nun in [mm]x_0[/mm] einsetzen soll. Kann man
> das hier aus der Aufgabenstellung heraus ablesen? Aus dem
> Intervall? Welche Grenze des Intervalls?
>  


Da es in der Aufgabe "mit Hilfe der Taylorformel" heißt,
ist davon auszugehen, daß Du die bekannte Taylorformel
für die Funktion [mm]e^{x}[/mm] heranziehen kannst.
Diese hat als Entwicklungspunkt [mm]x_{0}=0[/mm]


>
> Bei der Aufgabe soll ich ja die Funktion [mm]e^x[/mm] durch die
> Taylorformel soweit annähern das im obigen Intervall nur
> noch eine Abweichung von max. [mm]5 \cdot 10^{-5}[/mm] besteht. Wie
> gehe ich da nun vor? Soll man das ausprobieren wieviel
> Ableitungen da man da dann braucht? Oder gibt's eine
> elegantere Möglichkeit?
>  


Es gibt []Restgliedformeln.


Gruss
MathePower

Bezug
                
Bezug
Taylorformel: Frage (beantwortet)
Status: (Frage) beantwortet Status 
Datum: 17:43 Fr 17.06.2011
Autor: schubi


> Hallo bandchef,
>  
> > Bestimmen Sie für [mm]x \in [ 0,1 ][/mm] mit Hilfe der Taylorformel
> > einen Näherungswert für die Funktion [mm]e^x[/mm] der um
> > höchstens [mm]5 \cdot 10^{-5}[/mm] vom wahren Wert abweicht.
>  >  
> >
> > Die Taylor-Formel lautet ja so:
>  >  
> > [mm]T_n(x) = \sum_{k=0}^n {f^{(k)}(x_0) \over k!}(x-x_0) )^k = f(x_0) ) + \frac{f'(x_0) )}{1!}(x-x_0) ) + \frac{f''(x_0) )}{2!}(x-x_0) )^2 + \ldots + \frac{f^{(n)}(x_0) )}{n!}(x-x_0) )^n[/mm]
>  
> >  

> > Mir is in der Formel soweit alles klar, aber ich weiß
> > immer nicht was hier nun in [mm]x_0[/mm] einsetzen soll. Kann man
> > das hier aus der Aufgabenstellung heraus ablesen? Aus dem
> > Intervall? Welche Grenze des Intervalls?
>  >  
>
>
> Da es in der Aufgabe "mit Hilfe der Taylorformel" heißt,
>  ist davon auszugehen, daß Du die bekannte Taylorformel
>  für die Funktion [mm]e^{x}[/mm] heranziehen kannst.
>  Diese hat als Entwicklungspunkt [mm]x_{0}=0[/mm]
>  

Verstehe ich nicht ganz... die bekannte Taylorformel hat doch den Entwicklungspunkt nicht vorgegeben bei 0 oder? Ich meine, hier ist doch verlangt, dass die Abweichung in dem Bereich von 0-1 an keiner Stelle größer ist als [mm]5 \cdot 10^{-5}[/mm].

Oder geht das so, dass man EP = 0 annimmt und dann mittels Restglied bestimmt, dass die Abweichung an Stelle 0 und an Stelle 1 nicht größer als [mm]5 \cdot 10^{-5}[/mm] ist und es damit gegeben ist, dass an den Stellen zwischen 0 und 1 ebenfalls keine größere Abweichung existiert?

Bezug
                        
Bezug
Taylorformel: Antwort
Status: (Antwort) fertig Status 
Datum: 17:51 Fr 17.06.2011
Autor: MathePower

Hallo schubi,

> > Hallo bandchef,
>  >  
> > > Bestimmen Sie für [mm]x \in [ 0,1 ][/mm] mit Hilfe der Taylorformel
> > > einen Näherungswert für die Funktion [mm]e^x[/mm] der um
> > > höchstens [mm]5 \cdot 10^{-5}[/mm] vom wahren Wert abweicht.
>  >  >  
> > >
> > > Die Taylor-Formel lautet ja so:
>  >  >  
> > > [mm]T_n(x) = \sum_{k=0}^n {f^{(k)}(x_0) \over k!}(x-x_0) )^k = f(x_0) ) + \frac{f'(x_0) )}{1!}(x-x_0) ) + \frac{f''(x_0) )}{2!}(x-x_0) )^2 + \ldots + \frac{f^{(n)}(x_0) )}{n!}(x-x_0) )^n[/mm]
>  
> >  

> > >  

> > > Mir is in der Formel soweit alles klar, aber ich weiß
> > > immer nicht was hier nun in [mm]x_0[/mm] einsetzen soll. Kann man
> > > das hier aus der Aufgabenstellung heraus ablesen? Aus dem
> > > Intervall? Welche Grenze des Intervalls?
>  >  >  
> >
> >
> > Da es in der Aufgabe "mit Hilfe der Taylorformel" heißt,
>  >  ist davon auszugehen, daß Du die bekannte
> Taylorformel
>  >  für die Funktion [mm]e^{x}[/mm] heranziehen kannst.
>  >  Diese hat als Entwicklungspunkt [mm]x_{0}=0[/mm]
>  >  
>
> Verstehe ich nicht ganz... die bekannte Taylorformel hat
> doch den Entwicklungspunkt nicht vorgegeben bei 0 oder? Ich
> meine, hier ist doch verlangt, dass die Abweichung in dem
> Bereich von 0-1 an keiner Stelle größer ist als [mm]5 \cdot 10^{-5}[/mm].
>  
> Oder geht das so, dass man EP = 0 annimmt und dann mittels
> Restglied bestimmt, dass die Abweichung an Stelle 0 und an
> Stelle 1 nicht größer als [mm]5 \cdot 10^{-5}[/mm] ist und es
> damit gegeben ist, dass an den Stellen zwischen 0 und 1
> ebenfalls keine größere Abweichung existiert?


Es ist richtig, daß der EP = 0 angenommen wird.
Dann wird das Restglied in dem betreffenden Intervall
abgeschätzt. Und diese Abweichung wird dann mit
[mm]5 \cdot 10^{-5}[/mm] verglichen.


Gruss
MathePower

Bezug
                                
Bezug
Taylorformel: Frage (beantwortet)
Status: (Frage) beantwortet Status 
Datum: 19:12 Fr 17.06.2011
Autor: schubi

Hm okay, aber eine Frage bleibt doch noch ...
Bei der Restgliedbestimmung (zumindest bei der nach lagrange) muss man eine Funktion ja n+1 mal ableiten um die Abweichung bis n zu erhalten, sehe ich das richtig? Da die e-Funktion ja beliebig oft abgeleitet werden kann, kann man bei dieser Aufgabe ja trotzdem nur durch ausprobieren herausbekommen, inwiefern die geforderte Toleranz noch überschritten oder schon eingehalten wurde.

Ich meine, wenn ich T1 von [mm] x^2 [/mm] bestimmte, kann ich R1 ja noch bestimmen, weil [mm] x^2 [/mm] ja n+1, sprich 2 mal differenzierbar ist.

Hier fällt mir das Fehlerabschätzen leicht. Bei der e-Funktion hingegen geht das ja nicht ...

Hoffe ich konnte mich verständlich ausdrücken :)

LG

Bezug
                                        
Bezug
Taylorformel: Antwort
Status: (Antwort) fertig Status 
Datum: 19:30 Fr 17.06.2011
Autor: MathePower

Hallo schubi,

> Hm okay, aber eine Frage bleibt doch noch ...
>  Bei der Restgliedbestimmung (zumindest bei der nach
> lagrange) muss man eine Funktion ja n+1 mal ableiten um die
> Abweichung bis n zu erhalten, sehe ich das richtig? Da die
> e-Funktion ja beliebig oft abgeleitet werden kann, kann man
> bei dieser Aufgabe ja trotzdem nur durch ausprobieren
> herausbekommen, inwiefern die geforderte Toleranz noch
> überschritten oder schon eingehalten wurde.
>  
> Ich meine, wenn ich T1 von [mm]x^2[/mm] bestimmte, kann ich R1 ja
> noch bestimmen, weil [mm]x^2[/mm] ja n+1, sprich 2 mal
> differenzierbar ist.
>  
> Hier fällt mir das Fehlerabschätzen leicht. Bei der
> e-Funktion hingegen geht das ja nicht ...


Die Ableitung der Funktion [mm]e^{x}[/mm] ist die Funktion selbst.
Insofern kannst Du die n+1. Ableitung im Intervall [mm]\left[0,1\right][/mm]
leicht abschätzen.


>
> Hoffe ich konnte mich verständlich ausdrücken :)
>  
> LG


Gruss
MathePower

Bezug
                
Bezug
Taylorformel: Mitteilung
Status: (Mitteilung) Reaktion unnötig Status 
Datum: 19:43 Fr 17.06.2011
Autor: Al-Chwarizmi


> Da es in der Aufgabe "mit Hilfe der Taylorformel" heißt,
>  ist davon auszugehen, daß Du die bekannte Taylorformel
>  für die Funktion [mm]e^{x}[/mm] heranziehen kannst.
>  Diese hat als Entwicklungspunkt [mm]x_{0}=0[/mm]


Hallo,

da die Entwicklungsstelle [mm] x_0 [/mm] in der Aufgabe nicht vor-
gegeben ist, darf man sie wohl frei wählen.

Die Wahl [mm] x_0=0 [/mm] macht zwar die Rechnung am einfachsten.

Eine andere Wahl, zum Beispiel [mm] x_0=0.5 [/mm] , ergibt aber
möglicherweise eine Reihe, die weniger Glieder braucht,
um die gewünschte Genauigkeit zu erzielen.

LG    Al-Chw.

Bezug
        
Bezug
Taylorformel: Vorschlag
Status: (Antwort) fertig Status 
Datum: 21:52 Fr 17.06.2011
Autor: HJKweseleit


> Die Taylor-Formel lautet ja so:
>  
> [mm]T_n(x) = \sum_{k=0}^n {f^{(k)}(x_0) \over k!}(x-x_0) )^k = f(x_0) ) + \frac{f'(x_0) )}{1!}(x-x_0) ) + \frac{f''(x_0) )}{2!}(x-x_0) )^2 + \ldots + \frac{f^{(n)}(x_0) )}{n!}(x-x_0) )^n[/mm]

Die jeweilige Ableitung von f(x) = [mm] e^x [/mm] ist wieder [mm] e^x. [/mm]

Wenn du nun 0 als Entwicklungspunkt einsetzt, bekommst du bei jeder Ableitung für [mm] f^{(k)}(x_0) [/mm] den einfachen Wert [mm] e^0 [/mm] = 1 heraus. Nämest du einen anderen Entwicklungspunkt [mm] x_0, [/mm] so müsstest du immer [mm] e^{x_0} [/mm] berechnen - was aber blödsinnig wäre, denn du gehst doch davon aus, dass du [mm] e^x [/mm] noch gar nicht berechnen kannst und gerade deswegen die Reihenentwicklung machst....

Somit ergibt sich mit [mm] x_0 [/mm] = 0:

[mm]T_n(x) = \sum_{k=0}^n {1 \over k!}x^k = 1 + \frac{1}{1!}x + \frac{1}{2!}x^2 + \frac{1}{3!}x^3 + \ldots + \frac{1}{n!}x^n [/mm].

Das weggelassene Restglied entsprich dem Fehler, den du machst. Es hat den Wert (Lagrange)

[mm] \frac{f^{(n)}(\xi)}{(n+1)!}x^{n+1} [/mm] mit [mm] \xi \in [/mm] [0|1], also

[mm] \frac{e^{\xi}}{(n+1)!} x^{n+1}. [/mm] Leider braucht man nun doch einen Wert von [mm] e^{\xi}, [/mm] aber man muss nur abschätzen:

Im Intervall [0|1] hat [mm] e^{\xi} [/mm] maximal den Wert [mm] e^1 [/mm] = e<3 und [mm] x^{n+1} [/mm] maximal den Wert [mm] 1^1=1. [/mm] Somit ist der Rest maximal [mm] \frac{3}{(n+1)!}. [/mm] Dieser Wert bleibt [mm] <5*10^{-5}, [/mm] wenn [mm] (n+1)!>\frac{3}{5*10^{-5}}=60000 [/mm] ist. Damit kannst du n zu n=8 errechnen.


Bezug
                
Bezug
Taylorformel: Mitteilung
Status: (Mitteilung) Reaktion unnötig Status 
Datum: 23:29 Fr 17.06.2011
Autor: Al-Chwarizmi


> Nämest du einen anderen Entwicklungspunkt
> [mm]x_0,[/mm] so müsstest du immer [mm]e^{x_0}[/mm] berechnen - was aber
> blödsinnig wäre, denn du gehst doch davon aus, dass du
> [mm]e^x[/mm] noch gar nicht berechnen kannst und gerade deswegen die
> Reihenentwicklung machst....


Hallo HJK,

Mit [mm] x_0=0.5 [/mm] ist es natürlich leicht, [mm] e^{x_0}=\sqrt{e} [/mm] zu berechnen.
Oder sollen wir annehmen, dass nicht einmal der Wert
von e und die Quadratwurzelfunktion zur Verfügung
stehen ?
Wie schon gesagt, ist der Ansatz mit [mm] x_0=0 [/mm] natürlich
rechnerisch einfacher, hat aber den (kleinen) Nachteil,
dass man dann eventuell bei Taylor ein oder zwei Glieder
mehr braucht, um über das ganze Intervall die gewünschte
Genauigkeit zu erreichen.

LG   Al

Bezug
                
Bezug
Taylorformel: Frage (beantwortet)
Status: (Frage) beantwortet Status 
Datum: 15:20 Sa 18.06.2011
Autor: bandchef

Hallo!

Ich hab nun auch den Ausdruck stehen:

$60000 < (n+1)!$

Was ich aber hier jetzt nicht verstehe ist, wie ich hier nun n isolieren soll, damit ich auf das Ergebnis von 8 komme... Mich stört quasi die Fakultät am rechten Ausdruck...

Wie macht man das?

Bezug
                        
Bezug
Taylorformel: probieren
Status: (Antwort) fertig Status 
Datum: 15:25 Sa 18.06.2011
Autor: Loddar

Hallo bandchef!


Da es zur Fakultät keine explizite Umkehrfunktion gibt, bleibt hier nur Probieren.


Gruß
Loddar


Bezug
                                
Bezug
Taylorformel: Frage (beantwortet)
Status: (Frage) beantwortet Status 
Datum: 15:27 Sa 18.06.2011
Autor: bandchef

Hm, erstmal danke für deine Antwort. Aber was versteht man unter probieren?

Bezug
                                        
Bezug
Taylorformel: probieren
Status: (Antwort) fertig Status 
Datum: 15:28 Sa 18.06.2011
Autor: Loddar

Hallo!


Berechne z.B. $6!_$ , $7!_$ usw.


Gruß
Loddar


Bezug
                                        
Bezug
Taylorformel: Frage (beantwortet)
Status: (Frage) beantwortet Status 
Datum: 15:29 Sa 18.06.2011
Autor: bandchef

Ah, ich glaube ich weiß jetzt was gemein ist. Für n=7, also (7+1)! = 40320. Für n=8, also (8+1)!= 362880.

Somit folgt n=8.

Bezug
                                                
Bezug
Taylorformel: Antwort
Status: (Antwort) fertig Status 
Datum: 15:35 Sa 18.06.2011
Autor: MathePower

Hallo bandchef,

> Ah, ich glaube ich weiß jetzt was gemein ist. Für n=7,
> also (7+1)! = 40320. Für n=8, also (8+1)!= 362880.
>  
> Somit folgt n=8.


[ok]


Gruss
MathePower

Bezug
                                                
Bezug
Taylorformel: Antwort
Status: (Antwort) fertig Status 
Datum: 19:42 Sa 18.06.2011
Autor: Al-Chwarizmi


> Ah, ich glaube ich weiß jetzt was gemein ist. Für n=7,
> also (7+1)! = 40320. Für n=8, also (8+1)!= 362880.
>  
> Somit folgt n=8.


Das würde also bedeuten, dass du durch die Rest-
gliedabschätzung dazu kommst, dass man das
Taylorpolynom vom Grad 8 (entwickelt bei [mm] x_0=0) [/mm]
nehmen soll, damit der Fehler für alle x von 0 bis 1
unterhalb [mm] 5*10^{-5} [/mm] bleibt.

Man kann aber nachrechnen, dass effektiv auch
schon das Taylorpolynom [mm] T_7 [/mm] genügen würde.
Durch die Restgliedabschätzung (nach Lagrange)
erhält man also noch nicht das "optimale"
Ergebnis.
Würde man das Taylorpolynom an der Stelle [mm] x_0=0.5 [/mm]
(statt bei [mm] x_0=0) [/mm] entwickeln, würde effektiv sogar
[mm] T_5 [/mm] genügen ! Dies erreicht man allerdings auch
nicht mittels Restgliedabschätzung, sondern durch
tatsächliches Nachrechnen.
Setzt man aber etwa [mm] x_0:=0.52 [/mm] , so führt auch
schon Restgliedabschätzung zu n=5 (übereinstim-
mend mit dem effektiv nötigen Wert).
Natürlich möchte ich damit jetzt nicht etwa
propagieren, bei jeder Taylor-Approximation einen
solchen Aufwand zu betreiben. Die einfache Rechnung
mit [mm] x_0=0 [/mm] spricht immer noch für diese Wahl, obwohl
dann das Taylorpolynom zwei Glieder mehr als
minimal notwendig enthalten muss.

LG    Al-Chw.


Bezug
Ansicht: [ geschachtelt ] | ^ Forum "Funktionen"  | ^^ Alle Foren  | ^ Forenbaum  | Materialien


^ Seitenanfang ^
www.englischraum.de
[ Startseite | Forum | Wissen | Kurse | Mitglieder | Team | Impressum ]